5
$\begingroup$

I am looking at the series $X_{1,1},$$X_{2,1}, X_{2,2}$ $X_{3,1},X_{3,2},X_{3,3}$ $\dots$ of independent r.v's with $p_n:=P(X_{n,k}=1)=n^{-\frac{1}{4}}$ and $q_n:=P(X_{n,k}=0)=1-n^{-\frac{1}{4}}$. So they are Bernoulli-distributed.

I would like to know if$S_n:=\frac{\sum_{k\leq n}(X_{n,k}-E(X_{n,k}))}{Var(\sum_{k\leq n}X_{n,k}) }$ converges weakly, for $n\rightarrow \infty$.

One can observe that for every $n$ the sums $\sum_{k\leq n}X_{n,k} (=:Y_n)$ are $B(n,p_n)$ distributed. One gets

  • $E(X_{n,k})=p_n$,
  • $E(Y_n)=np_n$,
  • $Var(Y_n)=np_nq_n$.

So it is $S_n=\frac{Y_n-np_n}{np_nq_n }$.

The standard CLT can't be applied because the $Y_n$ have different winning-probabilities $p_n$. Also $Y_n$ does not converge to a Poisson-distributed r.v. because $np_n=n^{\frac{3}{4}}$ is not constant.

In which way can I apply the CLT?

  • 0
    A *very* similar question was asked within the last week on MathOverflow. Multiple different approaches were given in the answers.2012-02-21

1 Answers 1

2

I assume your definition of $S_n$ wants a square root in the denominator; otherwise it converges to 0.

You want the Lindeberg-Feller central limit theorem. See Theorem 3.4.5 of R. Durrett, Probability: Theory and Examples (4th edition).